Mean Value Theorem for integrals

Click For Summary
The discussion centers on proving the Mean Value Theorem for integrals using the Mean Value Theorem for derivatives and the Fundamental Theorem of Calculus. It establishes that if a function f is continuous on [a, b], then there exists a number c in [a, b] such that the integral of f from a to b equals f(c) multiplied by the length of the interval (b - a). The proof shows that since F(x) is continuous and differentiable, the derivative at some point c gives the average value of f over the interval. A point of confusion arises regarding the location of c, as the book states c can be at the endpoints a or b, even though F is differentiable only in the open interval (a, b). Ultimately, the discussion clarifies that while c must be in (a, b) for the derivative, the theorem allows for c to be at the boundaries in a broader context.
bigplanet401
Messages
101
Reaction score
0

Homework Statement



Prove the Mean Value Theorem for integrals by applying the Mean Value Theorem for derivatives to the function

<br /> F(x) = \int_a^x \, f(t) \, dt<br />

Homework Equations


[/B]
Mean Value Theorem for integrals: If f is continuous on [a, b], then there exists a number c in [a, b] such that

<br /> \int_a^b \, f(x) \, dx = f(c) (b - a) \, .<br />

Mean Value Theorem for derivatives, Fundamental Theorem of Calculus.

The Attempt at a Solution



Since the function f is continuous on [a, b], F(x) is continuous on [a, b] and differentiable on (a, b) by the FTC. The MVT for derivatives says

<br /> F(b) - F(a) = F^\prime (c) (b - a) \quad \text{for} \, c \in (a, b) \, .<br />

By FTC, F'(c) = f(c). Then f(c) (b - a) = F(b) - F(a). So

<br /> f(c) = \frac{F(b) - F(a)}{b - a} = \frac{1}{b - a} \int_a^b \, f(x) \, dx <br />

again by FTC.

What I don't understand is why the book says c can be equal to a or b...F is differentiable on the open interval (a, b), not [a, b]. But the book says the MVT for integrals permits c to lie anywhere in the closed interval [a, b].
 
Physics news on Phys.org
You derived a slightly stronger result - there has to be a "c" in (a,b). That implies the weaker result of the existence of a "c" in [a,b].
 
Question: A clock's minute hand has length 4 and its hour hand has length 3. What is the distance between the tips at the moment when it is increasing most rapidly?(Putnam Exam Question) Answer: Making assumption that both the hands moves at constant angular velocities, the answer is ## \sqrt{7} .## But don't you think this assumption is somewhat doubtful and wrong?

Similar threads

  • · Replies 12 ·
Replies
12
Views
1K
  • · Replies 11 ·
Replies
11
Views
2K
Replies
3
Views
2K
  • · Replies 14 ·
Replies
14
Views
2K
  • · Replies 3 ·
Replies
3
Views
2K
  • · Replies 26 ·
Replies
26
Views
3K
Replies
1
Views
2K
  • · Replies 2 ·
Replies
2
Views
1K
Replies
2
Views
1K
  • · Replies 7 ·
Replies
7
Views
2K